Foundations chapter 5,6,7,8,9,39,40,41

Ace your homework & exams now with Quizwiz!

Identify physician-initiated interventions: a. Administer antibiotic intravenously twice a day. b. Enable progressive ambulation as tolerated. c. Check vital signs four times a day. d. Provide preadmission teaching.

ANS: A It is very important for nurses to understand their scope of practice. Some interventions require a medical order. These types of interventions are physician-initiated interventions.

Nursing interventions for the patient who suffers from stress incontinence include kegel exercises. surgical interventions. bowel retraining. intermittent catheterization.

ANS: A Kegel exercises also are known as pelvic floor exercises. They improve muscle tone in the pelvic floor, which helps to prevent stress incontinence.

The physician has ordered 1000 mL of D5NS to infuse over 6 hours. The IV tubing has a drop factor of 10 gtts/min. Calculate the flow rate in cc/mL and gtts/min. Round to the nearest whole number for each calculation: ___________ mL/hr; ___________ gtts/min

ANS: 167; 28 1000mL/6 hours = 166.6 or 167 mL/hr 1000mL × 1 hour_____ × 10 gtts = 10,000 = 27.7 or 28 gtts/min 6 hours 60 minutes 1 mL 360

At 0900, the nurse hangs an IV of 1000 mL D5LR to infuse at 125 mL/hr. What time will the nurse need to hang a new bag of IV fluid? Provide your answer in military time: _____ hours.

ANS: 1700 1000mL/125mL = 8 hours. If the IV was hung at 0900, it will be infused 8 hours later, which is 1700 or 5:00 PM.

Which statement illustrates the most measurable outcome indicator? a. Demonstrates dressing change b. Shares innermost thoughts c. Understands instructions d. Shows personal remorse

ANS: A "Demonstrates dressing change" is a measurable outcome indicator. Outcome identification, added by the ANA in 1991 as a specific aspect of the nursing process, involves listing behaviors or observable items that indicate attainment of a goal. The other options are not measurable as written.

The hospice nurse believes the nursing diagnosis chronic sorrow is significant in the recovery process of patients recently experiencing a loss. What is required to support the addition of new nursing diagnoses to the NANDA-I taxonomy? a. Clinical research and data collection b. Changes in patient status and life experience c. Anecdotal nursing experiences d. Patient requests

ANS: A Clinical research, documenting the study findings of nurses who practice using the nursing process, is required to support the addition of new nursing diagnoses to the NANDA-I taxonomy. Nursing diagnoses are developed through comprehensive research and data collection to support the eventual confirmation of actual nursing diagnostic statements.

The nurse is inserting an indwelling catheter into a male patient. While initially passing the catheter through the urethra, resistance is met. What action should the nurse take next? Notify the primary care provider to place a coudé catheter. Straighten the penis and attempt to progress the catheter again. Remove the catheter and insert one with a smaller lumen. Inflate the balloon and wait for urine passage.

ANS: A Coudé catheters are a special type of double-lumen, indwelling catheters that are slightly stiff and bent at the end, allowing the catheter to pass more easily through a partially constricted urethra. They are used mostly in men experiencing prostate enlargement or BPH. Coudé catheters may need to be placed using a metal wire introducer. Placement using an introducer typically is performed by a physician or the patient's urologist, to avoid damaging urethral tissue.

Inclusion of the patient in the planning process is significant due to the fact that a. inclusion of the patient in the planning increases chances of goal attainment. b. the patient is the only person who should be setting goals. c. goal attainment is dependent on the excellence of the nurses' planning with the patient. d. patient participation is nice to include but not necessarily required.

ANS: A Effective goals are mutually acceptable to the nurse, patient, and family and are appropriate in terms of nursing and medical diagnoses and therapy. Inclusion of the patient and the family will promote collaborative goal attainment. Including the patient provides opportunity for the nurse to teach and involve the patient and family in the steps toward goal attainment and allows the patient to understand why activities are necessary for progress to healing.

Which action should the nurse take 30 minutes after administering oral pain medication to a patient? a. Evaluate the effectiveness of the administered pain medication. b. Teach progressive relaxation strategies to relieve muscle tension. c. Assess the patient's coping skills to reduce expressed anxiety. d. Encourage the patient to read or watch TV to provide pain distraction.

ANS: A Evaluation focuses on the patient and the patient's response to nursing interventions and goal or outcome attainment. During the evaluation step of the nursing process, nurses use critical thinking to determine whether a patient's short- and long-term goals were met and desired outcomes were achieved.

Prioritization of nursing diagnoses requires the use of which of the following tools? a. Maslow's hierarchy of needs b. Consideration of the ABC's, airway, breathing, circulation c. Basic life support assessment tool d. Advanced life support assessment tool

ANS: A Maslow's hierarchy of needs is held as an evidence-based, practical way to categorize patient needs stated in nursing diagnostic statements. This hierarchy provides guidance for the prioritization of nursing diagnostic statements. All other options can be categorized by the hierarchy as well.

An example of implementation of evidenced practice by the nurse would be the nurse a. initiates a new policy protocol for the removal of c-collars and bed board restraints of the emergency department patient based on empirical research results. b. watched a news report on a new procedure for chest tube removal and implements the procedure on the patient needing chest tubes removed. c. saw a physician perform a manipulation for vertigo related to inner ear problems and decides to utilize the manipulation for the current patient experiencing vertigo. d. is assisting a physician with conscious sedation during a procedure and is asked to perform outside the nursing scope of practice.

ANS: A New research findings are continuously resulting in policy changes for nursing practice. The nurse should embrace the new policy and procedures based on empirical research results. The other options are dangerous practices for nurses, are outside their scope of practice, and/or are not based on sound research results.

. Which body is responsible for defining and disseminating information on nursing diagnoses? a. North American Nursing Diagnosis Association International b. International and American Nurses Association c. Individual State Boards of Nursing d. The Joint Commission

ANS: A Nursing diagnoses are established and revised biannually by NANDA International, Inc. (NANDA-I), a professional nursing organization that provides standardized language to identify patient problems and plan customized care.

Which type of question would be best for the nurse to use when trying to determine the extent of a patient's knowledge concerning a disease process? a. Open ended b. Direct c. Close ended d. Focused

ANS: A Open-ended questions are best utilized to elicit a narrative response. Direct, focused, or close-ended questions are effective tools to guide an interview process and obtain demographic or specific information rather than a breadth or depth of data.

The nursing instructor is teaching information about constipation in the elderly. Which statement from the student indicates a need for further instruction on this topic? Patients receiving tube feedings often experience constipation. Poor fluid intake and inability to eat a high-fiber diet often cause constipation. Patients with impaired mobility may experience constipation. Medications commonly taken by elders often contribute to constipation.

ANS: A Patients on tube feedings often experience diarrhea, not constipation.

The nurse has just received a postoperative patient to the floor postureteral stone manipulation. Choose the priority nursing diagnosis. Risk for urinary retention r/t general anesthesia and trauma to ureter Pain, acute r/t recent surgical procedure and verbalization of pain of 4 on scale 0-10 Risk for bleeding r/t surgical site injury Comfort, impaired r/t inability to urinate and verbalization "I am beginning to feel full"

ANS: A The combination of general anesthesia and possible trauma to the ureter places the patient at risk for urinary retention. The other options are secondary to the risk for urinary retention. The "Risk for urinary retention r/t general anesthesia and trauma to ureter" is early and more pressing than "Comfort, impaired r/t inability to urinate and verbalization 'I am beginning to feel full.'"

The nurse has provided home care instructions to the mother of a child who is being discharged after cardiac surgery. Which statement by the mother indicates a need for further instruction? a. We are going to her cousin's birthday party tomorrow! The whole family will be there! b. I will need to keep the incision clean and dry. c. Some mild exercise with periods of rest is best. d. I can administer the pain medicine as prescribed on the label every four hours.

ANS: A The mother's answer clearly supports that she does not understand the infection control prevention measures for her child for the next few weeks. The other options support her understanding of the care for her child and further teaching seems not needed at this time.

Which of the following is an important component in evaluating patient outcomes and the plan of care? a. Nursing judgment and critical thinking b. Communication with the interdisciplinary team c. Implementing every intervention d. Nursing attitude

ANS: A The nurse must use strong clinical decision making and critical thinking to revise, modify, or discontinue the plan of care.

Which of the following statements most accurately reflects the nursing process? a. Cyclical in nature and steps overlap b. Can be delegated to increase productivity c. Must be completed in an orderly sequence from beginning to end d. Should follow standard structure for all patients

ANS: A The nursing process is cyclical and ever-changing as the patient condition changes.

Which statement illustrates appropriate documentation following palpation? a. Abdomen soft, non-tender without distention b. Density noted over kidney margins bilaterally c. Reddened area 3 inches in diameter noted on left thigh d. Heart sounds distant over the mitral and tricuspid valves

ANS: A The only documentation statement indicating palpation is assessment of the abdomen. Density is assessed via percussion. Reddened areas are identified through inspection and heart sounds are assessed through auscultation.

. A patient with a history of diarrhea is seen in the clinic. Which nursing intervention is most essential to include in this patient's plan of care? Weighing the patient daily Encouraging a diet high in fiber Decreasing the patient's fluid intake Instructing the patient to increase protein in the diet

ANS: A The patient with diarrhea is susceptible to dehydration. Checking the patient's weight daily will monitor fluid status.

Which of the following would the nurse expect to be included in the plan of care for a patient receiving total parenteral nutrition (TPN)? Blood sugar levels are checked on a routine basis Maintaining NPO status Hourly urine output Vital signs every 4 hours

ANS: A The usual composition of TPN begins with a high glucose solution (usually 25% glucose) and an amino acid solution. Because of the high glucose content, patients with TPN may develop hyperglycemia. Frequent monitoring of blood glucose can detect this increase. TPN can be used when the patient is NPO but can also be used as a supplement when oral intake is inadequate. Hourly outputs and frequent vital signs are not required for TPN infusions.

Which organism is responsible for the majority of urinary tract infections in female patients? Escherichia coli Nesseria gonorrhea Candida albicans Haemophilus influenza

ANS: A Urinary tract infections (UTIs) are the result of bacteria in the urine. Infection occurs when bacteria from the digestive tract, usually Escherichia coli, invade the urethra and multiply.

Which of the following statements would be considered incorrect when transfusing packed red blood cells (RBCs)? (Select all that apply.) Adjust the infusion rate to ensure unit is infused within 6 hours. Begin an infusion of D5W prior to the packed RBCs. Obtain baseline vital signs, including temperature and pulse oximetry. Verify the patient ID and blood unit number with another nurse prior to administration.

ANS: A, B The unit must be infused within 4 hr after leaving the laboratory. Only normal saline should be used with whole blood or packed RBCs. Use of any other IV solution can cause lysis or destruction of the red blood cells.

Identify the situation where collaborative interventions could be implemented: (Select all that apply.) a. physical therapy b. home health care c. palliative care d. performing surgery

ANS: A, B, C Collaborative interventions require cooperation among health care professionals and unlicensed assistive personnel (UAP). Collaborative interventions include activities such as physical therapy, home health care, personal care, spiritual counseling, medication reconciliation, and palliative or hospice care. These types of interventions require consultation with other health care professionals or referrals to specialists or agencies for assistance. Performing surgery is outside the scope of practice for a professional nurse and is not a collaborative intervention.

Goals are set during the planning step and (Select all that apply.) a. are broad statements of purpose that describe the aim of nursing care. b. represent short- or long-term objectives. C. are realistic and measurable. d. are generated by the patient without the input of the nurse.

ANS: A, B, C Goals are broad statements of purpose that describe the aim of nursing care. Goals represent short- or long-term objectives that are determined during the planning step. Useful and effective goals have certain characteristics. They are mutually acceptable to the nurse, patient, and family. They are appropriate in terms of nursing and medical diagnoses and therapy. The goals are realistic in terms of the patient's capabilities, time, energy, and resources, and they are specific enough to be understood clearly by the patient and other nurses. They can be measured to facilitate evaluation. The nurse's input into this process is critical to developing reasonable goals and interventions. Without the nurse's guidance during this step, the goals and interventions may be too weak to promote the patient's success or too aggressive for the patient to achieve.

. Identify nurse-initiated interventions: (Select all that apply.) a. ordering heel protectors for patients susceptible to skin breakdown b. consultations with social workers c. preadmission teaching d. ordering pain medication

ANS: A, B, C The ability of nurses to enact independent interventions has expanded in recent years, allowing nurses to initiate care that they recognize as essential in meeting patient needs or preventing complications. Ordering heel protectors for patients susceptible to skin breakdown and initiating preventive measures (e.g., activity regimens, consultations with social workers, preadmission teaching) are often independent, nurse-initiated interventions.

Which action(s) should the nurse take during the termination phase of the patient interview? (Select all that apply.) a. Express appreciation for the patient's participation. b. Review key assessment findings that were noted. c. Validate information covered with the patient. d. Allow the patient to add additional insights.

ANS: A, B, C, D The termination phase includes all of the actions mentioned. During the termination phase, interaction should be drawn to a logical conclusion and information should be reviewed, clarified, and verified prior to initiating care. Appreciation for the patient's participation should be acknowledged and the patient should be encouraged to add any additional information or thoughts that come to mind prior to ending the interaction.

Standards of care guide practice through (Select all that apply.) a. prudent performance of the nursing process. b. accountability. c. universal standards. d. development of the patient care plan.

ANS: A, B, D Professional nurses have more accountability now than at any other time in the history of nursing. Standards of care are met through the prudent performance of the nursing process. The planning step is critical for guiding patient care to acceptable patient outcomes. Standards of care are determined by governmental and nongovernmental committees and may be defined as local standards or national standards. The planning step provides concrete evidence to support the accuracy and pertinence of the patient care plan. Conscientious practice of the nursing process in development of the patient care plan ensures that professional standards of care are being met.

Which of the following findings would indicate effectiveness of fluid replacement for a patient admitted with dehydration? (Select all that apply.) Blood urea nitrogen - 18 mg/dL Pulse - 82 Blood pressure - 140/90 Urine specific gravity - 1.033 24-hour fluid balance - +200

ANS: A, B, E Blood urea nitrogen will be elevated with dehydration and return to normal levels with hydration. Normal pulse rate and positive fluid balance also indicate adequate fluid levels. An increased urine specific gravity is an indication of dehydration while an increased blood pressure can indicate fluid volume excess.

Prior to discharge, the nurse teaches the patient the proper techniques for applying an ostomy pouch. When evaluating the teaching, the nurse observes the patient apply a new ostomy pouch without cleansing the skin underneath. What action(s) should the nurse implement following this patient's return demonstration? (Select all that apply.) Repeat the demonstration to show the patient how to clean the ostomy site. Document that the patient performed the initial return demonstration accurately and safely. Offer positive reinforcement regarding the need to cleanse the site to prevent skin breakdown below the appliance. Discharge the patient with written instructions and illustrations that demonstrate the correct procedure. Notify the health care provider that a repeat demonstration of the ostomy appliance procedure is needed.

ANS: A, C, D The initial return demonstration was not performed accurately, and since it is the nurse's responsibility to complete the needed teaching, the health care provider does not need to be notified. Discomfort and damage to the skin can result from not washing the site; therefore, the nurse should repeat the demonstration, emphasizing the importance of cleansing. Positive reinforcement and the provision of written instructions are valuable teaching strategies.

1. Which of the following are components of delegation? (Select all that apply.) a. Assigning the correct task b. Assigning planning in the nursing process c. Having the LPN contact the physician for orders d. Using correct supervision to the delegate e. Assigning a task under the right circumstances

ANS: A, D, E The National Council of State Boards of Nursing has provided the Five Rights of Delegation to support safe delegation of patient care: right task, right circumstance, right person, right direction or communication, and right supervision or evaluation. The nursing process cannot be delegated. It is not in the scope of practice of the LPN to take physician orders. LPNs do not function independently.

6. Which long-term goal is written correctly? a. Patient will remain afebrile throughout hospitalization. b. Patient will return to professional sports activities within 6 months. c. Nurse will prevent bone infection through antibiotic therapy for 3 weeks. d. Patient will demonstrate accurate use of crutches without assistance before discharge from emergency room.

ANS: B "Patient will return to professional sports activities within 6 months" is a correctly written long-term goal. Goals that are achievable within an immediate time frame of less than approximately one week are short-term goals, whereas goals that will take more time to achieve—weeks to months—are long-term goals. All short- and long-term goals must be (1) patient focused, (2) realistic, and (3) measurable.

What should be the focus of all nursing interventions? a. Early hospital discharge for patients b. Providing patient-centered care c. Reduction of health care spending d. Delegating appropriate nursing care

ANS: B All patients are required to have unique, patient-centered plans of care designed to meet their specific needs.

. The nurse seeks assistance from the speech therapist on a patient's case to determine the patient's ability to swallow food. Which care technique is utilized here? a. Indirect communication b. Collaboration c. Delegation d. Assistive contribution

ANS: B Collaboration entails using expertise of health care professionals to pool resources and knowledge to provide quality care.

Which of the following is an example of collaboration? a. The nurse receiving orders from a physician b. The nurse and physical therapist creating an ambulation schedule for the patient c. The nurse arranging for discharge instructions to be provided to the patient and family d. The nurse providing the patient with a video on insulin injections

ANS: B Collaboration involves teamwork to accomplish a patient goal. The nurse and physical therapist creating an ambulation schedule can maximize the efforts and effects of the activity.

. What type of assessment is most appropriate for a patient newly admitted to the hospital for intermittent loss of vision in the left eye? a. Emergency b. Complete c. Focused d. Triage

ANS: B In this case, a complete assessment including examination of the cranial nerves is necessary upon admission to the acute care setting. Focused or clinical assessments will follow throughout the patient's hospitalization and at follow-up outpatient visits. There is no indication that the admission requires an emergency assessment or that triage to classify treatment priorities among patients is needed.

A patient presents to the emergency room with chest pain. Which of the following is the priority nursing intervention? a. Administer acetaminophen immediately. b. Provide oxygen via nasal cannula as ordered by the physician. c. Provide emotional support. d. Prepare the patient for emergency surgery.

ANS: B It is important for the nurse to prioritize using Maslow's hierarchy of needs.

Which of the following would be an inappropriate intervention for a patient with the nursing diagnosis of "Impaired Physical Mobility"? a. Use pressure relieving devices on bed and chair. b. Promote independence in performing all activities of daily living. c. Reinforce safety precautions with the patient and family. d. Perform active and passive range of motion three times daily.

ANS: B Promoting independence in all activities for a patient with mobility concerns presents a safety risk.

. The Nursing Outcomes Classification (NOC) is used for what purpose in the planning step? a. State whether the outcomes are appropriate for the implementation phase b. Provide the list of standardized nursing sensitive outcome indicators c. Be utilized for risk for nursing diagnoses d. Only apply to illness-related nursing diagnoses

ANS: B The Nursing Outcomes Classification index is specifically designed for nursing sensitive outcome indicators. Outcome indicators are criteria by which goal attainment is observed or measured.

The most important source in data collection is/are a. nursing literature. b. the patient. c. medical records. d. family members.

ANS: B The best source of information about the patient is the patient, not the family, the medical record, or nursing literature.

The charge nurse of a cardiac unit running code situation is practicing what type of planning? A. Initial patient planning B. Acute patient planning C. Discharge patient planning D. Maintenance patient planning

ANS: B The condition of the patient requires immediate performance of the steps of the nursing process quickly in this lifesaving action by the nurse. The nurse performs many types of planning based on the needs of the patient. A code situation required immediate and acute care planning.

A nurse admits a patient to the cardiac care unit following the placement of a cardiac stent. Which step of the nursing process does the nurse do first? a. Planning b. Assessment c. Evaluation d. Implementation

ANS: B The nurse first assesses the patient. Collecting an extensive health history and completing a thorough head-to-toe physical assessment are typically required when a patient is admitted to a hospital or seeking health care from a primary care provider for the first time. This information provides a baseline for future reference. Shorter, focused assessments are conducted by the nurse routinely throughout hospitalization or during repeated clinic visits to assess a patient's change of status.

Average urine pH is 4 6 7 9

ANS: B Urine normally is slightly acidic, with an average pH of 6. Urine with a pH of 4 is very acidic. A pH of 7 is neutral, and a pH of 9 is very alkaline.

Patients from which generation would be most comfortable with the nurse using electronic resources for health screening? a. Baby boomers b. Generation X c. Millennials d. Veterans

ANS: C Millennials are typically very computer literate or technology natives. Of the generations listed, millennials would be the most comfortable with accessing electronic resources for health screening.

The nurse is preparing to insert an IV catheter with an intermittent infusion device (IID) into an elderly woman for medication administration. Which of the following considerations would be incorrect? Insert the IV catheter into nondominant hand/arm. Use a 16- or 18-gauge over-the-needle catheter. Release the tourniquet before attaching the IID. Flush the IID with 2 to 3 mL normal saline after insertion

ANS: B With IV catheters, the higher the number, the smaller the gauge. For an elderly person who needed IV access for medication administration, a smaller gauge such as a 22 or 24 would be more appropriate. 16- or 18-gauge needles are used with high volume IV infusions.

The patient is to receive potassium 20 mEq every morning. Which of the following orders would the nurse question? (Select all that apply.) Potassium 10 mEq capsules. Administer 2 capsules PO. Potassium 15 mEq/5 mL liquid. Administer 10 cc of liquid PO. Potassium 2 mEq/1 mL solution. Add 10 cc to 1000 cc Lactated Ringers; infuse at 50 mL/h. Potassium 5 mEq/1 mL solution. Administer 4 cc IV over 10 minutes.

ANS: B, D 10 mL of liquid is equivalent to 30 mEq of potassium, which is higher than the ordered dose. Potassium should never be administered as an IV push medication.

Which statement by the nurse best describes health history assessment? a. "The first patient interview is the best source of all essential health history data." b. "When health history data is updated, patient information collected earlier is no longer useful." c. "Collection of health history information is ongoing and methodical throughout patient interaction." d. "Gathering health history data is best accomplished in a random, relaxed fashion as topics arise."

ANS: C A patient's health history is continuously evolving. Therefore, collection of health history data is ongoing and organized to avoid omission of critical information. Every patient interaction provides an opportunity to collect essential health history information. Older health history data are useful for reference regardless of how old the information may be. Health history information should be collected systematically in an established format to address essential topics and avoid accidental omissions.

Which statement is a correctly written example of an actual nursing diagnosis? a. Impaired memory related to patient complaint of becoming confused with the time change b. Risk for injury related to stumbling when walking as evidenced by patient report of occasional difficulty playing basketball c. Activity intolerance related to imbalance between oxygen supply and demand as evidenced by dyspnea on exertion and significant drop of oxygen saturation from 98% to 88% with activity d. Ineffective health maintenance as evidenced by inability to complete activities of daily living related to lack of familial support system

ANS: C Actual nursing diagnoses are written with three parts, whereas risk nursing diagnoses and health-promotion nursing diagnoses contain only two parts. Three-part nursing diagnosis statements include (1) the patient's identified need or problem (i.e., NANDA-I nursing diagnostic label), (2) the etiology or underlying cause (i.e., related to [r/t]), and (3) signs and symptoms (i.e., as evidenced by [AEB] or as manifested by [AMB]). Sleep Deprivation related to frequent sleep interruption as evidenced by patient complaint of diarrhea 10 times throughout the night and feeling fatigued is an example of a three-part nursing diagnosis statement. LO: 5.4

The clustering of data is significant to the nursing diagnoses step because clustering of data will a. show the nurse assessment is complete for this patient. b. move the nurse toward accurate planning for the symptoms in clustered data. c. group the data of similar problems and aid in accurate nursing diagnosis identification. d. organize the data for clear assessment so further assessment can occur.

ANS: C After collecting and reviewing all of the assessment data, the nurse looks for patterns and related data to support specific nursing diagnoses. This process is referred to as clustering data. Clustering involves organizing patient assessment data into groupings with similar underlying causes. The nurse looks for cues among the data that support the diagnosis of a problem. Clustering data is not associated with assessment. Symptoms are not the only data clustered. Data are clustered during diagnosis, not assessment.

A patient is admitted to the nursing unit with numbness and tingling in the right hand, pain in the cervical spine, and occasional loss of consciousness. Into which functional health pattern would the nurse organize this data? a. Self-perception and self-concept b. Coping and stress tolerance c. Cognition and perception d. Activity and exercise

ANS: C All of the patient's assessment findings are related to neurological functioning, which should be organized under the area of cognition and perception. Concerns regarding identity and body image would be documented under the health pattern of self-perception and self-concept. Stress tolerance and support system information would be recorded in the coping and stress management area. Assessment findings related to the cardiac, respiratory, and musculoskeletal systems are documented as part of the activity and exercise functional health pattern.

Which of the following IV solutions is considered hypertonic? Lactated Ringers D5W D5 0.45% NS 0.9% normal saline

ANS: C D5 0.45% NS is considered hypertonic because the osmolarity is greater than 290 mOsm/L. The other fluids are considered isotonic.

2. Which of the following is a direct care intervention? a. Reviewing the most recent clinical results from the laboratory b. Collaborating with social services regarding patient discharge plans c. Performing patient education regarding use of an incentive spirometer d. Obtaining medical records from a previous admission

ANS: C Direct care interventions are completed directly with the patient.

Documentation is a component of which part of the nursing process? a. Assessment b. Planning c. Implementation d. Evaluation

ANS: C Implementation includes performing and documenting nursing interventions.

The statement "ongoing collection of data" best describes which phase of the nursing process? a. Planning b. Evaluation c. Assessment d. Implementation

ANS: C Keeping the five steps of the nursing process in mind, a nurse conducts ongoing assessment (data collection) as a patient's condition changes and modifies the patient's plan of care on the basis of those findings.

A nurse is caring for an 80-year-old patient who is receiving bumetanide (a loop diuretic) for hypertension. The nurse notes that the patient admits to taking bisacodyl (Dulcolax) daily to stimulate her bowels. The nurse should assess the patient for possible symptoms of hypoglycemia. hypoparathyroidism. hypokalemia. hypocalcemia.

ANS: C Loop diuretics act on the loop of Henle to block reabsorption of sodium and potassium and are considered potassium-wasting diuretics. Daily use of laxatives such as bisacodyl can lead to increased potassium loss through stool.

. Which information gathered during assessment is considered to be subjective data? a. The client's urine is dark and foul-smelling. b. The patient's 24-hour urine output is 1800 mL. c. The patient indicates pain and burning are present when urinating. d. The patient is taking an antibiotic for a urinary tract infection.

ANS: C Patient verbalizations are subjective data. The rest of the options are pieces of data observed by the nurse and are considered objective data.

To best determine the patient's competency in changing an ostomy appliance, what should the nurse ask the patient to do? Verbalize the procedure. Identify the supplies needed. Perform the procedure. List the steps in the procedure

ANS: C Repeat performance is the best way to ensure competency.

Which discharge instruction should the nurse provide to the patient following a colonoscopy? Some discomfort and bleeding is normal postprocedure. Return to the emergency room if you experience mild abdominal cramping. Do not drive or operate heavy machinery for 12 hours postprocedure. Return to your normal bowel pattern immediately postprocedure.

ANS: C Since sedation is given for the procedure, the patient should not drive or operate heavy machinery.

A disoriented patient is admitted to the hospital accompanied by his spouse. From whom should the nurse collect subjective data on this patient? a. An experienced nurse on the unit b. The patient's medical record c. The patient's wife d. His physician

ANS: C Subjective data (i.e., symptoms) are spoken. Patients' feelings about a situation or comments about how they are feeling are examples of subjective data. Data shared by a source verbally are considered subjective. Subjective data may be difficult to validate because they cannot be independently and objectively measured. Subjective data are most often gathered during a patient interview or health history. Use of an interpreter may be necessary when the patient or family members speak a language unfamiliar to the nurse. Subjective data are typically documented in the patient's medical record as direct quotations; for example, "I didn't get much sleep last night" or "I've had diabetes since I was 10 years old."

The Nursing Interventions Classification (NIC) index is used for what purpose in the planning step? a. Provides guidance for selection of nursing interventions b. Is updated annually for accuracy c. Lists appropriate interventions for nursing diagnoses d. Is useful during the second step of the nursing process

ANS: C The Nursing Interventions Classification (NIC) is a comprehensive, research-based, standardized collection of interventions and associated activities. NIC provides nurses with multidisciplinary interventions linked to each NANDA-I nursing diagnosis and a corresponding NOC.

The nurse working in the patient discharge center is practicing what type of planning? Initial patient planning Acute patient planning Discharge patient planning Maintenance patient planning

ANS: C The needs of the patient at this particular time are related to discharge. The patient needs are not in the initial steps or the acute phase at this time.

What term best describes the nature of the nursing process? a. Static b. Linear c. Dynamic d. Predictable

ANS: C The nursing process is dynamic, changing over time in response to patients' individual needs. The dynamic, responsive nature of the nursing process allows it to be used effectively with patients in any setting and at every level of care, from the intensive care unit to outpatient wellness clinics.

What should be included in teaching for a patient who will be discharged with a prescription for a laxative? Calling the health care provider if nausea, vomiting, or abdominal pain occurs Continuing use of laxatives to encourage bowel evacuation Adding regular exercise, sufficient fluids, and regular defecation habits to his or her routine Knowing the difference between laxatives and cathartics

ANS: C The patient who is discharged on laxatives should still be instructed on the nonpharmacological methods to decrease constipation and promote normal bowel patterns. Laxatives are contraindicated in patients with nausea, vomiting, or undiagnosed abdominal pain. Ongoing use of laxatives is associated with harmful side effects, such as an increase in constipation and impaction, predisposition to colorectal cancer, dependency, and electrolyte imbalance and should not be encouraged. Knowing the difference between laxatives and cathartics will not help the patient in this case.

The patient is ordered an ultrasound to determine the size, shape, and location of the kidneys. The nurse knows that prior to the test the patient will be required to have a bowel cleansing enema. be checked for any allergies to shellfish. be required to drink a large amount of fluids before the test. have no pretest requirements.

ANS: D An ultrasound scan may be performed to assess the size, shape, and location of the kidneys. Ultrasound studies may be safely conducted in patients who have allergies to contrast media, because no radiation or contrast dyes are used. No patient preparation such as fasting or sedation is required.

A nurse is caring for an adult patient who has gastric suction following abdominal surgery. The patient tells the nurse that he has tingling in his fingers and toes and is feeling dizzy. Which acid-base imbalance is the patient most likely experiencing? Respiratory alkalosis Metabolic acidosis Respiratory acidosis Metabolic alkalosis

ANS: D In metabolic alkalosis, there is an excess of bicarbonate ions, which raises the pH above 7.45 and produces bicarbonate levels greater than 26 mEq/L. This occurs as a result of loss of gastric acids through vomiting or nasogastric suctioning, among other causes. Clinical manifestations include numbness and tingling in the fingers and toes.

Prior to identifying accurate nursing diagnoses, what action must be taken by the nurse? a. Reading the patient's history b. Setting realistic, measurable goals c. Comparing evidence-based practices d. Clustering related patient data

ANS: D In the diagnosis step, patient data are analyzed, validated, and clustered to identify patient problems. Each problem is then stated in standardized language as a specific nursing diagnosis to provide greater clarity and universal understanding by all care providers. Reading the patient's history is part of assessment. Goal setting and considering evidence-based practice are part of planning.

. A patient is experiencing acute renal failure. What is the most common cause of this critical illness? Hypovolemia Cardiogenic shock Nephrotoxic substances Urethral obstruction

ANS: D Inadequate flow or complete obstruction by anything (such as stones or tumors) that blocks both ureters and the bladder, or obstructs the urethra, can lead to an anuric state, resulting in acute or chronic renal failure.

A nurse admits a 5-year-old female to the postanesthesia unit following a tonsillectomy. The child is crying. What should be the nurse's first action? a. Tell the child that if she stops crying, her parents can be with her. b. Check to see what pain medication is ordered for the child. c. Notify the surgeon of the child's postoperative condition. d. Assess the child to determine why she is crying.

ANS: D Nurses assess the state of a patient's physical, psychological, emotional, environmental, cultural, and spiritual health to gain a better understanding of his or her overall condition.

. The relationship of the medical diagnosis to the nursing diagnosis is a. the medical diagnosis is embedded within the nursing diagnostic statement. b. nursing diagnoses are driven by/derived from the medical diagnosis. c. the medical diagnosis is not relevant to the nursing diagnosis. d. the medical and nursing diagnoses should complement each other.

ANS: D Nursing diagnoses take into consideration a patient's attitudes, strengths, and resources—not just the medical problems identified—which are critical for planning holistic, individualized care. Medical diagnoses are not written within the nursing diagnosis statements. Nursing diagnoses are not derived from medical diagnoses. There is relevance of nursing diagnoses to medical diagnoses.

Outcome indicators are a. broad statements that reflect the nursing diagnosis. b. established by the physician. c. used to evaluate the quality of nursing interventions. d. criteria by which goal attainment is observed or measured.

ANS: D Outcome indicators are criteria by which goal attainment is observed or measured. They are not broad statements that reflect the nursing diagnosis and they are not used to evaluate the quality of nursing interventions. Outcome identification, which was added by the American Nurses Association (ANA) in 1991 as a specific aspect of the nursing process, involves listing observable behaviors or items that indicate attainment of a goal. Nurses play a critical role in achievement of positive outcomes and in outcome identification.

7. What phrase best describes the essence of critical thinking? a. Understanding without conscious reasoning b. Providing care based on nursing experience c. Consulting with a primary care provider d. Seeking solutions to problems

ANS: D Seeking solutions to problems describes the essence of critical thinking. Paul (1988) describes critical thinking as a complex process during which individuals think about their thinking to provide clarity and increase precision and relevance in a specific situation while attempting to be fair and consistent.

Which statement is the best example of subjective, secondary data? a. Unlicensed assistive personnel reports patient's blood pressure is 138/84 b. Patient complains of extreme fatigue and dizziness when walking in the room c. Nurse states that the patient's chest x-ray has a shadow in the left upper lobe d. Spouse reports patient has been vomiting intermittently for the last 48 hours

ANS: D Subjective, secondary data is spoken information or symptoms shared by someone other than the patient. Blood pressure readings and chest x-ray findings are examples of objective data. Patient statements are primary, subjective data.

Which piece of assessment data may be accurately obtained during the observation phase? a. Pulse irregularity b. Slow capillary refill c. Elevated temperature d. Presence of body odor

ANS: D The nurse uses the senses of sight, hearing, and smell during the observation phase of assessment. The presence of body odor is the only patient data listed that can be accurately assessed during the observation phase. Pulse irregularity, slow capillary refill, and elevated temperature all require vital sign assessment or palpation.

Select the most appropriate goal for a patient experiencing diarrhea related to antibiotic use: The patient will return to previous elimination pattern. The patient will increase intake of grains, rice, and cereals. The patient will discontinue antibiotic use and contact the health care provider. The patient will increase fluid intake.

ANS: D The patient will increase fluid intake since diarrhea can lead to dehydration.

A patient is scheduled for an upper GI series. Which information is most important to obtain from him before the procedure? Allergy to lasix Last bowel movement Time the enema was administered Any difficulty swallowing

ANS: D The patient will need to drink barium for this x-ray; therefore, swallowing ability should be assessed prior to the start of the procedure.

The teaching plan for a patient with diarrhea should include which intervention? a. Drinking at least eight glasses of fluid each day b. Eating foods low in sodium and potassium c. Limiting the amount of soluble fiber in the diet d. Eliminating whole-wheat and whole-grain breads and cereal

Answer: a Diarrhea is associated with high risk for dehydration, so the patient should increase the fluid intake. The patient may need increased sodium and potassium intake owing to loss of these electrolytes in the frequent stools. Fiber will add bulk and help to form the stools so should be increased. Whole-grain products contain fiber.

The nurse has begun an infusion of fresh frozen plasma (FFP). Which symptom indicates an allergic reaction to the FFP? a. Respirations: 30/min b. Urine output: 50 mL/hr c. Heart rate: 62 beats/min d. Temperature: 39° C (102.2° F)

Answer: a Dyspnea and tachypnea are signs of a severe allergic reaction. The urine output is an adequate hourly amount. The heart rate is normal. An increased temperature usually indicates an infection or a febrile nonhemolytic reaction, but assessment requires knowing the patient's baseline temperature.

Which cue by a patient can be validated by laboratory and diagnostic test results? a. Deeply sighing with fatigue b. Bilateral crackles in the lungs c. Oxygen saturation of 98% on room air d. 2+ pitting edema of the ankles and feet

Answer: a A cue is a behavioral hint of a potential disease process or concern. In this case, the only cue is a deep sigh indicating fatigue. The level of fatigue can be verified by evaluating the patient's hemoglobin and hematocrit levels for anemia. Crackles, oxygen saturation, and pitting edema are all physical assessment findings, not cues.

What is the most significant problem that may result from improperly written nursing diagnostic statements? a. Lack of direction for formulating patient plans of care b. Omission of physician or primary care provider orders c. Combining of two unrelated patient concerns d. Increased team collaboration needs

Answer: a Accurate nursing diagnostic statements provide direction for the development of individualized plans of care. Orders are part of the patient's assessment data. Combining unrelated patient problems is a function of diagnostic development, not a result of an improperly written statement. Poorly written nursing diagnostic statements may or may not result in increased team collaboration.

Which activity is important to include in the plan of care for a client with a peripherally inserted central catheter (PICC)? a. Use sterile technique when changing the PICC dressing. b. Change the IV tubing every 72 hours. c. Take blood pressure in the arm with the PICC line. d.Use only macrodrip tubing with IV infusions through the PICC line

Answer: a Because a PICC enters the body through a peripheral vein and is threaded up to the superior vena cava, resting just outside the right atrium of the heart, strict sterile technique is used during insertion and care of PICCs to prevent entrance of bacteria into the line. PICC tubing is usually changed every 24 hours. Never take blood pressure in an arm with a PICC. Macrodrip or microdrip tubing can be used for infusions through a PICC.

A patient with an indwelling catheter reports a need to void. What is the priority intervention for the nurse to perform? a. Check to see if the catheter is patent. b. Reassure the patient that it is not possible to void while catheterized. c. Catheterize the patient again with a larger-gauge catheter. d. Notify the primary care provider (PCP).

Answer: a Checking the position and patency of the catheter first will determine whether the problem is mechanical or physiologic in nature. At times, the end of the catheter may become lodged up against the side of the bladder preventing the flow of urine into the tubing. Telling the patient that is impossible to void while catheterized is erroneous. Catheterizing the patient with a larger-gauge catheter is unnecessary at this point, as is contacting the PCP.

Which statement is an appropriately written short-term goal? a. Patient will walk to the bathroom independently without falling within 2 days after surgery. b. Nurse will watch patient demonstrate proper insulin injection technique each morning. c. Patient's spouse will express satisfaction with patient's progress before discharge. d. Patient's incision will be well approximated each time it is assessed by the nurse.

Answer: a Goals are to be patient-focused, realistic, and measurable. Only the first goal meets these three criteria.

Which situation indicates the greatest need for collaborative interventions provided by several health care team members? a. Hospice referral b. Physical assessment c. Activities of daily living d. Health history interview

Answer: a Hospice referral requires collaboration with many health care team members. Physical assessment and completion of a health history interview are independent nursing actions that can be performed by a nurse alone. Activities of daily living can be completed by patients independently or with the help of a nurse or unlicensed assistive personnel (UAP), requiring little collaboration among health care team members.

What should be the primary focus for nursing interventions? a. Patient needs b. Nurse concerns c. Physician priorities d.Patient's family requests

Answer: a Patient needs are always the primary focus of nursing interventions. Nursing concerns, physician priorities, and family requests can provide additional guidance in the development of a patient-centered plan of care.

1. What is the primary difference between a risk nursing diagnosis and an actual nursing diagnosis? a. Defining characteristics are not part of a risk diagnosis. b. There is no cause and effect relationship established. c. Defining characteristics are subjective in a risk diagnosis. There are no nursing interventions prescribed with a risk diagnosis

Answer: a Risk diagnoses do not have defining characteristics; actual and health-promotion nursing diagnosis statements have defining characteristics. Risk diagnoses do not establish a cause and effect, because they identify potential rather than existing problems. Risk diagnoses contain related or risk factors rather than defining characteristics, subjective or otherwise. Risk diagnoses, like actual diagnoses, have nursing interventions to address a patient's current or potential problem.

A patient reports feeling tired and complains of not sleeping at night. What action should the nurse perform first? a. Identify reasons the patient is unable to sleep. b. Request medication to help the patient sleep. c. Tell the patient that sleep will come with relaxation. d. Notify the physician that the patient is restless and anxious.

Answer: a When a patient shares a concern, the first action by the nurse is to assess potential reasons for the patient's problem. Depending on the underlying reason for the patient's inability to sleep, the nurse may then want to administer prescribed sleep medication, teach the patient some relaxation techniques, or discuss patient behaviors with the primary care provider.

1. What should the nurse consider before implementation of all nursing interventions? (Select all that apply.) a. Potential communication barriers b. Diverse cultural practices c. Scope of nursing practice d. Functional status of the patient e. Time of most recent shift change

Answer: a, b, c, d Cultural practices, functional status, communication barriers, and scope of practice influence whether an intervention should or may be implemented. Shift change time is not necessary to consider before implementation of most interventions.

What should be taken into consideration by the nurse when deciding on interventions to include in a patient's plan of care? (Select all that apply.) a. Patient's treatment preferences b. Cultural and ethnic influences c. Professional level of expertise d. Current evidence-based research e. Convenience to the nursing staff

Answer: a, b, c, d Patient treatment preferences, cultural and ethnic influences, the level of a nurse's professional expertise, and current evidence-based research should all be taken into consideration when planning care. The convenience to nursing staff should not be of concern.

Which factors should be taken into consideration by the nurse before and during a patient interview? (Select all that apply.) a. Distance between the chairs in which the nurse and patient are sitting b. Traditional treatments typically used by the patient to treat disease c. Gender preference for primary care providers d. Physical condition of the patient e. Music preference of the patient

Answer: a, b, c, d The first four factors are important for the nurse to consider when initiating or conducting a patient interview. The distance that is comfortable for personal interaction and gender preferences for care providers are affected by cultural and age norms. During the interview, it is an important aspect of assessment to ask patients about the treatments that they traditionally use in response to illness. Preferred treatments sometimes can be incorporated into care plans. The physical condition of patients affects their ability to answer questions during an interview. It may be necessary to break the interview process into short periods to accommodate the patient who is seriously ill. Music preference is irrelevant because there should not be music playing during the interview because it would be a distraction.

Which action is a part of the evaluation step in the nursing process? (Select all that apply.) a. Recognizing the need for modifications to the care plan b. Documenting performed nursing interventions c. Determining if nursing interventions were completed d. Reviewing whether a patient met their short-term goal e. Identifying realistic outcomes with patient input

Answer: a, d Determining whether a goal or outcome is met is part of the evaluation. Making sure interventions are completed and documenting them are part of implementation. Identifying outcome criteria is done during the planning stage of the nursing process.

Which nursing goal is written correctly for a patient with the nursing diagnosis of Risk for Infection after abdominal surgery? a. Nurse will encourage use of sterile technique during each dressing change. b. Patient's white blood count will remain within normal range throughout hospitalization. c. Patient's visitors will be instructed in proper hand washing before direct interaction with patient. d. Patient will understand the importance of cleaning around the incision with a clean cloth during bathing.

Answer: b A patient's white blood cell count is a laboratory test that is a measureable indicator of infection. The correct answer is also patient-focused and realistic. Encouraging the use of sterile technique by the nurse during each dressing change and instructing the patient's visitors in the proper handwashing technique before direct interaction with the patient are not patient-focused. The patient understanding the importance of cleaning around the incision with a clean cloth during bathing uses a nonmeasurable verb, which should be avoided when formulating patient goals.

A patient with an excessive alcohol intake has a reduced amount of antidiuretic hormone (ADH). Which symptom is the patient likely to exhibit? a. Dysuria b. Polyuria c. Oliguria d. Hematuria

Answer: b ADH acts to reabsorb water in the nephron. When ADH secretion is decreased, excess water is excreted, leading to increased urine output, or polyuria. The opposite of oliguria is occurring. Because decreased ADH does not have an effect on the patient's ability to void, dysuria is not a symptom. Decreased ADH does not cause blood in the urine.

If a patient is exhibiting signs and symptoms of each of the following nursing diagnoses, which should the nurse address first while planning care? a. Fatigue b. Acute Pain c. Knowledge Deficit d. Body Image Disturbance

Answer: b Acute Pain is the most urgent nursing diagnosis to address. Fatigue may be a result of the pain and may be alleviated if the patient's pain level is reduced. Body Image Disturbance and Knowledge Deficit can be treated only after the patient's pain level is at an acceptable level. Both diagnoses require teaching, during which the patient needs to concentrate. A person's ability to concentrate is affected by the pain level.

The nurse notices that a patient is becoming short of breath and anxious. Which of the following interventions is a dependent nursing action, requiring the order of a primary care provider? a. Elevating the head of the patient's bed b. Administering oxygen by nasal cannula c. Assessing the patient's oxygen saturation d. Evaluating the patient's peripheral circulation

Answer: b Before a nurse can legally administer oxygen to a patient, the method of delivery and amount must be ordered by the primary care provider. Elevating the head of the bed and assessing a patient's oxygen saturation and peripheral circulation are all independent nursing interventions.

The nurse is placing an indwelling catheter in a female patient. She inserts the catheter into the vagina. What is the next action for the nurse to implement? a. Collect a urine specimen and notify the PCP. b. Leave the catheter in place and insert a new catheter into the urethra. c. Remove the catheter from the vagina and place it into the urethra. d. Ask another nurse to attempt the catheterization of the patient.

Answer: b By leaving the first catheter in place in the vagina, the nurse can more accurately identify the urethra for insertion of the new catheter. This prevents misplacing the new catheter into the vagina during the second catheterization attempt. The catheter that was placed in the vagina is no longer sterile, so it should not be reused and should be discarded after the new catheter is properly placed. It is impossible to get a urine sample from the catheter placed in the vagina. Only after experiencing difficulty with proper placement of the new catheter may the nurse wish to ask for assistance from another nurse.

Which activity by the nurse best demonstrates part of the working phase of a patient interview? a. Summarizing previously discussed key topics b. Including selected family members in care planning c. Transferring care responsibilities to the home health nurse d. Verifying the name by which a patient prefers to be addressed

Answer: b Care planning takes place during the working phase of the nurse-patient interview. When a patient needs care assistance, it is important for family members who will be helping with the patient's care to be involved in the process. Verifying the name that a patient prefers to be called takes place during the orientation or introductory phase. Summarizing key topics covered in the interview and transferring care responsibilities take place in the termination phase.

A patient is scheduled for an intravenous pyelogram (IVP). Which piece of data would be most important to know before the procedure is carried out? a. Urinalysis showing negative results on testing for sugar and acetone b. History of allergies c. History of a recent thyroid scan d. Frequency of urination

Answer: b Contraindications tor IVP include an allergy to iodine, which is similar in nature to the contrast material injected during the intravenous pyelogram. Knowing this information would be critical to providing safe patient care. Frequency of urination may be an indication to perform an IVP; however, this is not critical to know before performing an IVP. The results of a urinalysis and history of a recent thyroid scan would not affect a scheduled IVP.

A 40-year-old patient complains of 4 days of frequent loose stools with abdominal cramping. What is the priority nursing diagnosis for this patient? a. Altered Skin Integrity b. Risk for Imbalanced Fluid Volume c. Acute Pain d. Self-Care Deficit: Toileting

Answer: b Diarrhea can cause dehydration with loss of fluids and electrolytes. There is no statement of problems with the skin, although this patient may be at risk for skin breakdown if the diarrhea continues. In addition, no self-care deficit is stated for this patient. Although the patient has experienced cramping and the pain needs to be addressed, the main consideration would be correction of any fluid and electrolyte problems, followed by determination of the cause of the diarrhea.

What is the primary purpose of the nursing diagnosis? a. Resolving patient confusion b. Communicating patient needs c. Meeting accreditation requirements d.Articulating the nursing scope of practice

Answer: b Each nursing diagnosis label identifies either a patient problem or need, which is its purpose. Resolving patient confusion, meeting accreditation requirements, and articulating the nurse's scope of practice are not related to the purpose of the nursing diagnostic process.

An indwelling catheter is ordered for a postoperative patient who is unable to void. What is the primary concern of the nurse performing the procedure? a. Teaching deep-breathing techniques b. Maintaining strict aseptic technique c. Medicating the patient for pain before the procedure d. Positioning the patient for comfort during the procedure

Answer: b It is most important to maintain strict aseptic technique while inserting an indwelling catheter to try to prevent a urinary tract infection. It is not necessary to medicate patients before urinary catheterization. Although comfortable positioning and deep breathing may help to relax the patient, this is not the primary concern.

A nurse in the emergency department is caring for an adult patient with numerous draining wounds from gunshots. The patient's pulse rate has increased from 100 to 130 beats/min over the past hour. For which imbalance should the nurse assess symptoms? a. Respiratory acidosis b. Extracellular fluid volume deficit c. Metabolic alkalosis d. Intracellular fluid volume excess

Answer: b The draining wounds indicate hypovolemia, or extracellular fluid volume deficit. As circulating blood volume decreases, the heart rate increases to maintain normal cardiac output. Respiratory acidosis and metabolic alkalosis do not have as a symptom a rapidly increasing pulse rate. Intracellular fluid volume excess causes pulmonary congestion and cerebral edema.

The nurse is assessing a patient with an indwelling catheter and finds that the catheter is not draining and the patient's bladder is distended. What action should the nurse take next? a. Notify the primary care provider (PCP). b. Assess the tubing for kinks and ensure downward flow. c. Change the catheter as soon as possible. d.Aspirate the stagnant urine in the catheter for culture

Answer: b The next action by the nurse should be to check the patency of the catheter tubing. At this point there is no need to aspirate any urine or call the PCP. The catheter should not be changed unless absolutely necessary, owing to the possibility of causing an infection.

Which action by the nurse is most appropriate during the orientation phase of the patient interview? a. Always position patients in a comfortable reclined position to ensure their comfort during questioning. b. Ask which name a patient prefers to be called during care to show respect and build trust. c. Quickly conduct a review of systems to determine the need for a complete or focused assessment. d. Begin with questions about intimacy and sexuality to address sensitive issues first.

Answer: b The nurse should provide a personal introduction and establish the name by which the patient wants to be called at the very beginning of the interview as part of the orientation phase. In most cases, the patient and the nurse should be seated at eye level during the interview portion of the assessment. Questions about intimacy and sexuality should be reserved for later in the interview to establish rapport before exploring potentially sensitive issues. A review of systems takes place during the working phase of the nurse-patient interview, just before initiation of the physical assessment.

1. A patient has just experienced a cardiac arrest on the unit. The nurse has implemented the acute care plan for management of code situations. What is the next step the nurse should take? a. Resume all interventions for previously identified nursing diagnoses. b. Perform the steps of the nursing process related to the patient's current condition. c. Seek physician input related to updating the nursing diagnosis statements. d. Evaluate the success of the acute care plan for management of the cardiac arrest.

Answer: b The patient's condition requires immediate performance of the lifesaving steps of the nursing process. All other answers are secondary actions. The nurse later resumes all interventions for previously identified nursing diagnoses and evaluates the success of the acute care plan for management of the cardiac arrest. Nurses do not seek the input of the physician for creation of nursing diagnoses.

A patient is being discharged from the hospital with a new ileostomy. The patient expresses concern about caring for the ostomy. Before hospital discharge, it is most important for the nurse to coordinate with which member of the health care team? a. Home care nurse b. Wound ostomy continence nurse c. Registered dietitian d. Primary care provider

Answer: b The wound ostomy continence nurse (WOCN) is the most important person to contact to schedule teaching sessions and follow-up care. This nurse specialist is certified in the treatment of patients who have a bowel or bladder diversion. Although team input is important, the contribution of the WOCN is paramount to help the patient achieve competence and comfort with self-care before discharge.

Which nursing instruction is correct when a urine specimen is collected for culture and sensitivity testing from a patient without a urinary catheter? a. Tell the patient to void and pour the urine into a labeled specimen container. b. Ask the patient to void first into the toilet, stop midstream, and finish voiding into the sterile specimen container. c. Instruct the patient to discard the first void and collect the next void for the specimen. d. Have the patient keep all voided urine for 24 hours in a chilled, opaque collection container.

Answer: b Urine specimens for culture and sensitivity testing must be collected in sterile containers using the clean-catch, midstream method whenever possible. All voided urine specimens should be collected directly into the specimen container, not transferred from another potentially contaminated vessel. Discarding the entire first void and saving urine in a chilled, opaque container are both procedures for conducting a 24-hour urine collection

Which nursing intervention is most important to complete before giving medication to a patient? a. Provide water to aid in the patient's ability to swallow the medication. b. Double-check the patient's allergies before giving the drug. c. Ask the patient to verify having taken the medication before. d. Place the patient in a side-lying position to prevent aspiration.

Answer: b Verifying patient allergies before administering medication is the most important intervention listed to ensure patient safety. Providing water may or may not be necessary, depending on the type of medication being administered. Although it is okay to ask a patient about having taken a medication previously, it is not routinely done or most important. It is preferable to have patients sit up while taking medication unless contraindicated.

Which intervention can the nurse initiate independently while providing patient care? (Select all that apply.) a. Ordering a blood transfusion b. Auscultating lung sounds c. Monitoring skin integrity d. Applying heel protectors e. Adjusting antibiotic dosages

Answer: b, c, d Auscultating lung sounds and monitoring skin integrity are both important aspects of basic patient assessment that are required independent nursing actions. Ordering and applying heel protectors is done independently by nurses to prevent skin breakdown on patient's confined to the bed. Ordering blood transfusions and adjusting antibiotic dosages are the responsibility of the patient's primary health care provider.

1. Which nursing diagnosis is appropriately written? (Select all that apply.) a. Risk for Infection related to elevated temperature and white blood count b. Readiness for Enhanced Relationship as evidenced by mutual respect verbalized by spouses and expressed desire for improved communication c. Noncompliance related to inability to access care as evidenced by failure to keep appointments, homebound status d. Risk for Bleeding with the risk factor of prolonged clotting time e. Chronic Pain related to osteoarthritis as manifested by verbalized postoperative discomfort.

Answer: b, c, d Readiness for Enhanced Relationship is a heath-promotion nursing diagnosis and is written with two sections: the label and the defining characteristics. Noncompliance is a nursing diagnosis that requires a related factor and defining characteristics. Risk for Bleeding requires at least one risk factor, which it has as it is written. Use of related factors in a risk nursing diagnosis is not the accepted NANDA-I format. The nursing diagnosis of Chronic Pain is incorrectly written because it includes a medical diagnosis and a related factor that is supportive of acute rather than chronic pain.

1. What signs and symptoms would the nurse appropriately cluster for a patient with extreme anxiety? (Select all that apply.) a. Denies any difficulty falling asleep b. Elevated pulse rate auscultated at 140 BPM c. Continuous foot tapping throughout intake interview d. Demonstrates how to give insulin self-injection without hesitation e. Patient states, "I feel nervous all the time, especially when I am alone."

Answer: b, c, e An elevated pulse rate, continuous toe tapping, and verbalizing nervousness are consistent with extreme anxiety and should be clustered together. Ease of falling asleep and being able to focus on a challenging task, such as giving an injection, are not indicative of a patient experiencing a high level of anxiety.

3. An alert, oriented patient is admitted to the hospital with chest pain. Who is the best source of primary data on this patient? a. Family member b. Physician c. Another nurse d. Patient

Answer: c Assessment begins at the moment the patient first interacts with the nurse. Redressing takes place at the end of the physical examination. Breathing during auscultation is part of the respiratory assessment, and sharing health history and demographic information takes place during the patient interview.

Which action by a patient marks the beginning of the physical assessment process? a. Redressing after a physical examination b. Breathing normally during auscultation c. Greeting the nurse in the examination room d.Sharing work environment information

Answer: c Assessment begins at the moment the patient first interacts with the nurse. Redressing takes place at the end of the physical examination. Breathing during auscultation is part of the respiratory assessment, and sharing health history and demographic information takes place during the patient interview.

When initiating a physical examination, which action should the nurse take first? a. Review of the patient's prior medical records b. Gather admission health history forms c. Assess the patient's vital signs d. Perform light and deep palpation for fluid

Answer: c Assessment of the patient's vital signs begins the physical examination aspect of the assessment process. This provides the nurse with baseline information about cardiac and respiratory function, pain level, and temperature. The nurse should review the patient's prior medical records before the interview or after the patient interaction to fill in gaps. Admission health history forms need to be gathered before initiating the interview, and abdominal palpation takes place about halfway through the head-to-toe physical examination.

The nurse is caring for a patient who needs to increase calcium in her diet but does not like milk. Which food should the nurse encourage the patient to consume? a. Cod b. Eggs c. Spinach d. Tomatoes

Answer: c Dark leafy vegetables such as spinach, kale, turnip greens, broccoli, Brussels sprouts, and cabbage are sources high in calcium.

Which action by the day-shift nurse provides objective data that enables the night- shift nurse to complete an evaluation of a patient's short-term goals? a. Encouraging the patient to share observations from the day b. Leaving a message with the charge nurse before shift change c. Documenting patient assessment findings in the patient's chart d. Checking with the pharmacist regarding possible drug interactions

Answer: c Documentation of assessment findings is the only objective form of data listed as an option that can support the night nurse in evaluating whether the patient achieved short-term goals. Patient observations are subjective in nature. Leaving a message with the charge nurse produces secondary subjective data, and checking for drug interactions is unrelated to the evaluation process.

9. Which nursing intervention is included for a patient experiencing diarrhea? a. Limiting fluid intake to 1000 mL/day b. Administering a cathartic suppository c. Increasing fiber in the diet d. Limiting exercise

Answer: c Fiber is encouraged in patients with diarrhea to add bulk to the stools. Fluid intake and exercise should be encouraged. Cathartics would not be used because they are strong laxatives used to soften the stool and evacuate the bowels.

Which action by the nurse would be most important in developing a patient-centered plan of care for an alert, oriented adult? a. Providing a written copy of care options to the patient and family b. Collaborating with the patient's social worker to determine resources c. Listening to the patient's concerns and beliefs about proposed treatment d. Engaging the patient's family, friends, or care providers in conversation

Answer: c It is most important to involve the patient in developing realistic, attainable, patient-centered plans of care. Involving others in care planning is secondary to involving the patient, unless the patient is cognitively impaired.

Which nursing action is critical before delegating interventions to another member of the health care team? a. Locate all members of the health care team. b. Notify the physician of potential complications. c. Know the scope of practice for the other team member. d. Call a meeting of the health care team to determine the needs of the patient.

Answer: c Knowing the scope of practice of the other team member is critical to understanding what is appropriate and safe to delegate to that person. It is unnecessary to locate or meet with all members of the health care team prior to delegation. Physicians are already aware of potential complications related to patient care.

Which resource is most helpful when prioritizing identified nursing diagnoses? a. Nursing Interventions Classification (NIC) b. Gordon's functional health patterns c. Maslow's hierarchy of needs d. Nursing Outcomes Classification (NOC)

Answer: c Maslow's hierarchy of needs and the airway, breathing, circulation (ABCs) of life support are the most helpful tools in identifying priorities of care. Functional health patterns is one method of organizing assessment data. NOC and NIC are resources for identifying outcomes and interventions to include in a patient's care plan after priorities have been established.

On what premise is a nursing diagnosis identified for a patient? a. First impressions b. Nursing intuition c. Clustered data d.Medical diagnoses

Answer: c Nursing diagnoses emerge from groupings of clustered data collected during the assessment phase of the nursing process. The nurse documents the patient's medical diagnosis as one piece of data, which may be clustered with others to support a nursing diagnosis. Data collected from a nurse's intuition and first impressions may also be listed in the patient's assessment findings as long as they are objectively recorded without prejudice and are not judgmental in nature.

Which entry in a patient's electronic health record best indicates the need for a nurse to gather secondary rather than primary subjective data? a. Complaining of chest pain b. Apical pulse 110 c. Comatose d.Difficulty swallowing

Answer: c Primary data are obtained from the patient directly. A patient who is comatose is unable to speak and therefore unable to share subjective, primary data. A patient complaining of chest pain has already shared primary, subjective data. A patient with an apical pulse of 110 who is alert or one who has difficulty swallowing may still be able to contribute subjective information to the data collection.

1. Which phrase best represents a related factor in an actual nursing diagnosis? a. Unsteady gait requiring the assistance of two people b. Redness and swelling around the incision site c. Ineffective adaptation to recent loss d. Patient complaint of restlessness

Answer: c Related factors are broad statements that indicate the cause for the defining characteristics, which are signs or symptoms identified from collecting the patient's data. Redness and swelling, unsteady gait, and complaint of restlessness are specific defining characteristics that would be clustered with other data to support the existence of an actual or health-promotion nursing diagnosis.

The nurse is assessing the intravenous (IV) site in the right forearm and notices the area around it is cool, swollen, firm, and tender to touch. Which complication is most likely occurring? a. Infection b. Speed shock c. Infiltration d. Phlebitis

Answer: c The area around an infiltration is cool, swollen, firm, and tender to touch. In cases of infection and phlebitis, the skin is warm to touch. Speed shock produces systemic symptoms, such as dizziness, headache, and irregular heart rate.

What action should the nurse take regarding a patient's plan of care if the patient appears to have met the short-term goal of urinating within 1 hour after surgery? a. Consult the surgeon to see if the clinical pathway is being followed. b. Discontinue the plan of care, because the patient has met the established goal. c. Monitor patient urine output to evaluate the need for the current plan of care. d. Notify the patient that the goal has been attained and no further intervention is needed.

Answer: c The nurse should evaluate the need to continue or discontinue a plan of care if a patient has met a short-term goal. It is unnecessary to consult the surgeon unless there is a concern. Discontinuing the care plan may be premature, and the decision needs to be evaluated before taking action. The patient's intake and output will continue to be monitored throughout hospitalization, not just for 1 hour after surgery.

What is the purpose of the nursing process? a. Providing patient-centered care b. Identifying members of the health care team c. Organizing the ways nurses think about patient care d. Facilitating communication among members of the health care team

Answer: c The nursing process is the methodology used to "think like a nurse." Providing patient-centered care and enhancing communication among health team members is facilitated through the use of care plans. Collaborating with rather than identifying members of the health care team is part of many plans of care.

A patient is scheduled for a colonoscopy. After preprocedure teaching by the nurse, the patient demonstrates understanding when he makes which statement? a. "I can have coffee the morning of the procedure." b. "I should drink a red sports drink the day before to stay hydrated." c. "I should drink clear liquids for 2 days before the procedure." d. "I will be able to drive home immediately after the procedure."

Answer: c The patient will be on a clear liquid diet for 1 to 3 days before the procedure. The patient should not eat or drink anything immediately before the procedure. Drinks with red or purple dye are contraindicated because they could interfere with the exam findings. Patients are given medications during the procedure that alter the sensorium and therefore need to have someone else drive them home.

What self-care measure is most important for the nurse to include in the teaching plan for a patient who will be discharged with a urostomy? a. Change the appliance before going to bed. b. Cut the wafer 1 inch larger than the stoma. c. Cleanse the peristomal skin with mild soap and water. d. Use firm pressure to attach the wafer to the skin.

Answer: c The peristomal area can be washed by the patient using warm water and mild soap as needed and routinely at bath time. The collection device typically has a face plate to ensure a good fit and prevent leakage of urine. These appliances are changed less frequently than before bed each night, and neither a widely cut wafer or firm pressure is needed for their application.

Which task may the registered nurse safely delegate to unlicensed assistive personnel without prior intervention? a. Ambulating a patient with ataxia and new right sided paresthesia b. Feeding a patient with cerebral palsy who recently aspirated c. Transporting a patient to the hospital entrance for discharge d. Administering prescribed programmed medications

Answer: c Transporting the stable patient for discharge can be delegated immediately to UAP. A patient with new neurologic symptoms needs to be assessed before being ambulated. Patients who have recently choked need to be evaluated for their ability to swallow before being fed. Administering medication is not within the UAP's scope of practice and can never be delegated to UAP.

1. Which intervention would be most important for the nurse to include in a patient's care plan if the patient is unable to complete activities of daily living without becoming fatigued? a. Instruct the patient to shower and shave simultaneously b. Discourage the patient from bathing while hospitalized c. Encourage the patient to rest between bathing activities d. Ask the patient's spouse to assist with all bathing

Answer: c When patients are unable to complete their personal care without fatigue, it is best to encourage them to rest between activities. All patients should be encouraged to wash during hospitalization and to complete as much of their personal care as independently as possible. Patients who tire easily should not be encouraged to shower and shave simultaneously but should space out personal care while seated.

Which notation is most appropriate for the nurse to include in a patient's chart regarding evaluation of the goal, "Patient will ambulate three times daily in the hallway before discharge without shortness of breath (SOB)"? a. Goal not met; patient states he is tired. b. Goal not met; patient ambulated three times in room. c. Goal met; patient ambulated three times in the hallway. d. Goal met; patient ambulated three times in the hallway without SOB.

Answer: d Option d is the only notation that indicates whether the goal was met and how all of the outcome criteria were attained.

If the nurse discovers that a patient's right elbow is swollen and painful during a physical examination, which action should the nurse take next? a. Apply ice to decrease swelling and reduce pain b. Percuss the area to determine the presence of fluid c. Perform passive range of motion to promote flexibility d. Inspect the patient's left elbow to compare its appearance

Answer: d A major aspect of assessment is checking for symmetry. If an abnormality is observed on one side of a patient's body, the next step in the assessment is to compare that area with the other side. Applying ice is premature until the assessment is complete and an underlying cause of the swelling and pain is understood. Percussion is not indicated for assessment of a swollen elbow. Performing passive range of motion is not appropriate before identifying an injury or disease and determining its extent.

A patient has reported a 2-kg (4.4-lb) weight gain over the past 3 days. Which factor should the nurse question? a. Protein intake b. Potassium intake c. Calorie intake d. Sodium intake

Answer: d A weight gain of 2 kg in 3 days suggests fluid retention. Increased sodium intake leads to increased fluid retention. Although it is important to ask the patient about intake of all nutrients, the other options cannot cause this much weight gain in 3 days

While performing an abdominal assessment on an unconscious patient, the nurse notes presence of an ostomy. The fecal output is liquid in consistency, with a pungent odor, and the stoma is located in the upper right quadrant of the abdomen. What type of ostomy does the patient have? a. Descending colostomy b. Ureterostomy c. Ileostomy d. Ascending colostomy

Answer: d An ascending colostomy meets the description of fecal output of liquid consistency and with a pungent odor, as well as location of the stoma in the upper right quadrant of the abdomen. Descending colostomies produce increasingly formed stool. An ileostomy will produce liquid stool but with less odor because enzyme activity is not present. Ureterostomies drain urine, not stool.

Which line of questioning by the nurse best represents an appropriate approach to the review of systems aspect of the assessment process? a. "What do you do for a living? Can you describe your work environment?" b. "Is there a family history of heart disease, cancer, high blood pressure, or stroke?" c. "When was your last annual physical? What immunizations did you receive at that time?" d. "Do you have any chest tightness, shortness of breath, or difficulty breathing while exercising?"

Answer: d During a review of systems, the patient is asked questions about each body system to determine the level of functioning. Asking about work-related information, family history, and immunizations is accomplished during the collection of health history data before initiating the review of systems.

Which direct-care intervention would be most effective in helping a patient cope emotionally with a new diagnosis of cancer? a. Reassessing for changes in the patient's physical condition b. Teaching the patient various methods of stress reduction c. Referring the patient for music and massage therapy d. Encouraging the patient to explore options for care

Answer: d Encouraging the patient to explore options for care empowers the patient to have some control over the situation and to be actively involved in care planning. It is a form of informal counseling. Reassessment and teaching are not immediately indicated at this time. Although referring a patient with a new cancer diagnosis may be helpful, it is an indirect care intervention.

The nurse knows that the teaching for a patient who was recently diagnosed with constipation has been effective if the patient's meal request specifies which food choice? a. Hot dog on a bun b. Grilled chicken c. Tuna sandwich on white bread d. Spinach salad with dressing

Answer: d Green leafy vegetables are high in fiber. None of the other options are high in fiber but could be modified by using whole-grain products.

2. A patient comes to the emergency department complaining of nausea and vomiting. What should the nurse ask the patient about first? a. Family history of diabetes b. Medications the patient is taking c. Operations the patient has had in the past d. Severity and duration of the nausea and vomiting

Answer: d In an emergent situation, the nurse initially focuses on the patient's chief complaint to determine its cause. Before initiating care, the nurse gathers information on the other topics. LO: 5.4

A patient discusses his job stress and family relationships with the nurse during his health history interview. In which organizational framework is this type of data likely to be recorded most extensively? a. Body systems model b. Physical assessment model c. Head-to-toe assessment model d. Functional health patterns model

Answer: d Job stress and family relationships data will only be recorded extensively when using the Functional health patterns model. The functional health patterns model is holistic in its approach. The body systems model and head-to-toe assessment model focus on physical rather than psychological or emotional concerns. All three models listed are ways to organize physical assessment findings.

Which statement illustrates a characteristic of goals within the care planning process? a. Goals are vague objectives communicating expectations for improvement. b. Short-term goals need not be measurable, unlike long-term goals. c. Goal attainment can be measured by identifying nursing interventions. d.Long-term goals are helpful in judging a patient's progress.

Answer: d Long-term goals are very useful in determining patient progress. Both short-term and long-term goals need to be measurable. Goal attainment is based on patient actions, not nursing actions.

1. Which statement best describes the relationship of medical diagnoses and nursing diagnoses? a. Medical diagnoses are imbedded in nursing diagnoses. b. Nursing diagnoses are derived from medical diagnoses. c. Medical diagnoses are not relevant to nursing diagnoses. d. Medical diagnoses may be interrelated to nursing diagnoses.

Answer: d Nursing diagnoses consider the underlying etiology, needs, potential concerns, and patient response to a patient's medical diagnosis, so the two types of diagnoses are interrelated. Medical diagnoses are not imbedded or derived from medical diagnoses because that would limit the scope of assessment and care that is provided for patients. Nurses consider the medical diagnosis as one aspect of concern when identifying an actual or potential health problem and the patient's response, so medical diagnoses are relevant, but not the focus of nursing diagnoses.

What symptom is most likely to be exhibited by the patient who complains of voiding small amounts of urine in relation to his fluid intake? a. Nocturia b. Polyuria c. Anuria d. Oliguria

Answer: d Oliguria is reduced urine volume. Nocturia is excessive urination at night. Polyuria is an excessive amount of urine excreted each day, and anuria is excretion of 50 to 100 mL or less of urine each day.

A 65-year-old female patient is a two-pack-a-day cigarette smoker with a history of chronic obstructive pulmonary disease (COPD). What is the interpretation of her arterial blood gas values (pH 7.34, PCO2 55, PO2 82, HCO3- 32)? a. Partially compensated respiratory alkalosis b. Uncompensated metabolic acidosis c. Uncompensated respiratory alkalosis d. Partially compensated respiratory acidosis

Answer: d Patients with COPD tend to have chronic carbon dioxide retention. The patient is slightly acidotic (i.e., arterial pH below 7.35) with a higher than normal partial pressure of carbon dioxide (PCO2). which is inverse and therefore respiratory in nature. The compensatory response to respiratory acidosis is buffering, as indicated by the higher than normal bicarbonate (HCO3-) level. The increase in bicarbonate only partially shifts the pH toward normal, but partial compensation prevents the acid-base imbalance from becoming life-threatening. The kidneys will continue to compensate in an attempt to bring the pH into the normal range.

Which action would the nurse undertake first when beginning to formulate a patient's plan of care? a. List possible treatment options b. Identify realistic outcome indicators c. Consult with health care team members d. Rank patient concerns from assessment data

Answer: d Prioritizing or ranking patient needs precedes the identification of outcome indicators, consulting with team members, or consulting with interdisciplinary team members.

What is the primary purpose of quality improvement? a. Recognizing the need to discipline employees violating policies b. Preventing patient injury that may contributor to the death of others c. Increasing institutional profits to support further scientific research d. Enhancing current practices to improve patient outcomes and care

Answer: d Quality improvement focuses on improving processes. This has many benefits and is not primarily directed at preventing the death of patients, providing discipline, or making money.

1. What is the most important reason for nurses to use a standardized taxonomy such as NANDA-I? a. Insurance documentation b. Professional autonomy c. Role delineation d. Patient safety

Answer: d Safety is the most important reason for using standardized language to communicate patient's needs and information. Using the same definitions of terms helps nurses and other health care professionals interpret the information. Helping with insurance documentation, supporting professional autonomy, and clarifying the nursing role in patient care are uses for NANDA-I taxonomy, but they are not the most important.

1. What is the most important action for a nurse take in order to have a new nursing diagnosis considered for inclusion in the NANDA-I taxonomy? a. Share concerns with the nurse manager on the nursing unit b. Offer alternative care for a patient and family members c. Discuss how to address patient needs with physicians d. Provide evidence-based research to support nursing care

Answer: d Supporting a suggestion for a new nursing diagnostic label with research is required for consideration by NANDA-I. Sharing concerns, providing alternative care, and advocating for patients are all a part of the nursing role, but are not the most important part of having a diagnosis considered for inclusion in the NANDA-I taxonomy.

The nurse is assigned the care of a patient for whom a cleansing enema has been ordered. What information is most important for the nurse to know before administration of the enema? a. The proper way to position the patient b. Signs and symptoms of intolerance to the procedure c. Vital signs before the procedure d. History of surgery of the anus or rectum

Answer: d The most important item in preadministration assessment data is a history of surgery to the anus or rectum, which may contraindicate enema administration. The nurse needs to know the proper patient position for an enema and must observe for signs of intolerance to the procedure, but these are done during the procedure. Vital signs are not routinely obtained before an enema.

A nurse caring for a hospitalized patient is told in the shift change report that the patient's laboratory results are sodium = 140 mEq/L; potassium = 4.1 mEq/L; calcium = 9.5 mg/dL; and magnesium = 3.4 mEq/L. Which abnormal level will the nurse report to the primary care provider? a. High sodium level b. Low potassium c. Low calcium level d. High magnesium level

Answer: d The only abnormal value is the magnesium. Normal magnesium levels for adults range from 1.8 to 3.0 mg/dL. The other values are within normal limits.

A female patient has had frequent urinary tract infections. Which statement by the patient indicates that the nurse's teaching on prevention has been effective? a. "I will limit my fluid intake to 40 ounces per day." b. "I will use bubble bath when bathing." c. "I will wait to wear my tight jeans until after my urine is clear." d."I will wipe from the front to back after voiding

Answer: d Wiping the female perineal area from front to back after voiding is crucial in the prevention of microorganisms, which lead to infection, being transferred from the rectum or vagina to the urethral meatus. Limiting fluid intake, using bubble bath, and wearing tight-fitting clothing all may contribute to the promotion of urinary tract infections, rather than their prevention.

When emptying a patient's catheter drainage bag, the nurse notes that the urine appears to be discolored. The nurse understands that what factors may change the color of urine? (Select all that apply.) a. Taking the urinary tract analgesic phenazopyridine (Pyridium) b. A diet that includes a large amount of beets or blackberries c. An enlarged prostate or kidney stones d. High concentrations of bilirubin secondary to liver disease e.Increased carbohydrate intake

Answers: a, b, c, d Urine may appear orange when a patient is taking phenazopyridine. Urine can appear red or pink with a diet including beets or blackberries and if blood is present in the urine, which may be secondary to an enlarged prostate or kidney stones. Urine often has a brownish appearance when liver disease such as hepatitis or cirrhosis is present.

1. Which action does the nurse need to take before determining the type(s) of nursing diagnoses that are applicable to a patient? (Select all that apply.) a. Thoroughly review the patient's medical history b. Analyze the nursing assessment data to determine whether information is complete c. Outline an individualized plan of care to address each concern d. Consider potential complications to which the patient is susceptible e. Evaluate how the patient has responded to treatment

Answers: a, b, d Before determining the types of nursing diagnoses that are appropriate for a patient, the nurse must review and analyze all of the patient's data, including the medical history, for completeness and accuracy. Considering the vulnerability of a patient to potential complications permits the nurse to identify the need for risk nursing diagnoses. Outlining an individualized plan of care takes place during the planning stage of the nursing process after the nursing diagnoses have been identified. Evaluation of a patient's response to treatment is part of the evaluation stage of the nursing process.

If the nurse chooses the Nursing Outcome Classification (NOC), Appetite (1014) for a chemotherapy patient, which outcome indicators would be acceptable for evaluation of goal attainment? (Select all that apply.) a. Expressed desire to eat b. Report that food smells good c. Use of relaxation techniques before meals d. Preparation of home-cooked meals for self and family e. Uses nutritional information on labels to guide selections

Answers: a, b, d Sharing a desire to eat, reporting that food smells good, and preparing meals are indications of an increased appetite. Although relaxation techniques may decrease anxiety associated with eating, they do not indicate an increase in appetite. Reading nutrition labels is unlikely to increase a person's appetite.

To prevent constipation in an inactive patient, which early interventions should the nurse implement? (Select all that apply.) a. Stool softener administration b. Enema administration c. Increasing the fiber in the diet d. Increasing physical activity e. Increasing fluid intake

Answers: a, c, d, e Administering stool softeners, increasing the fiber and fluids in the diet, and increasing physical activity are all early interventions to prevent constipation. Although used to treat constipation, an enema would not be an early intervention for prevention.

When administering a cleansing enema, which techniques should the nurse use? (Select all that apply.) a. Assist the patient to a left side-lying (Sims) position. b. Perform hand hygiene and apply sterile gloves. c. Add room-temperature solution to enema bag. d. Lubricate 2 to 4 cm (1 to 2 inches) of tip of rectal tube with lubricating jelly. e. Raise container, release clamps, and allow solution to flow to fill tubing. f. Hang solution bag 45 to 60 cm (18 to 21 inches) above anus and instill rapidly. g. Clamp tubing after solution is instilled

Answers: a, e, g The patient should be assisted to the left side-lying (Sims) position. The container release clamps must be released and the solution allowed to flow for fill the tubing. After the solution is instilled, the tubing should be clamped. Gloves for this procedure do not need to be sterile. Solution should be warmed to slightly warmer than body temperature (or 100° to 105° F) to prevent cramping. The tip of the rectal tube should be lubricated 6 to 8 cm (3 to 4 inches). If the enema bag is hung too high and the solution is instilled too rapidly, cramping may occur.

For a patient with a nursing diagnosis of Fluid Volume Deficit, the nurse is alert to which signs and symptoms? (Select all that apply.) a. Hypertension b. Flushed skin c. Dry mucous membranes d. Weak, thready pulse e. Pale yellow urine

Answers: b, c, d Depending on the severity of fluid volume deficit, the patient may have hypotension. The skin is flushed and dry, and the pulse is weak and thready. Hypertension occurs with fluid volume overload. For patients with fluid volume deficit, the urine is dark yellow and concentrated.


Related study sets

Human Origins and the Neolithic Revolution

View Set

Lecture 19 - Adaptations to living in freshwater environments

View Set

Chapter 12: Somatic Symptom Disorders (Exam 2)

View Set

L7 : Dehydration of 2-methylcyclohexanol

View Set

Computer Concepts for End Users: Unit 11

View Set